mr 1 2007 solutions

Upload: andre-luiz-aulas-matematica-fisica

Post on 04-Jun-2018

219 views

Category:

Documents


0 download

TRANSCRIPT

  • 8/13/2019 MR 1 2007 Solutions

    1/43

    Solutions for Mathematical Reflections 6(2006)

    Juniors

    J31. Find the least perimeter of a right-angled triangle whose sides and

    altitude are integers.

    Proposed by Ivan Borsenco, University of Texas at Dallas

    Solution by Jos Alejandro Samper Casas, Colegio Helvetia de Bogota,Colombia

    Solution. The answer for the least possible perimeter is 60. It holdsfor a right-angled triangle (15, 20, 25),whose altitude is 12.

    Letx, y,zbe a Pythagorean triple withzthe hypotenuse and leth bethe altitude of the triangle. Let d = gcd(x,y,z) be the greatest commondivisor of x,y,z. We have that x = d a , y = d b and z = d c fora,b,c a primitive Pythagorean triple. Now calculating the area of thetriangle in two ways we obtain that h = xy

    z = abd

    c . Using the fact that

    gcd(ab,c) = 1,we getc|dwhich tells us thatc

    d,since both are positive

    integers. Because the perimeter is equal to d(a+b+c) c(a+b+c),we can minimize it be taking c = d. Then the altitude of a right-angledtriangle having sides ca,cb,c2 (with c2 the hypotenuse) is ab and is aninteger.

    We know that a,b,c is a primitive Pythagorean triple if and only ifthere exist m, n Z+ such that gcd(m, n) = 1,m n (mod 2) m > n >0that satisfy a = m2 n2, b = 2mn, c = m2 +n2. Replacing in (1), wenotice that all we need to find is the minimum value of

    p= (m2 +n2)(2m2 + 2mn).

    Clearlym > n >0, therefore m 2 andn 1. Thusp (22 + 1)(2 22 + 2 2 1) = 60.

    Now the triangle with sides (15, 20, 25) satisfies all the conditions of theoriginal problem and its perimeter is 60. The problem is solved.

    Also solved by Ashay Burungale, India

    Mathematical Reflections 1 (2007) 1

  • 8/13/2019 MR 1 2007 Solutions

    2/43

    J32. Leta and b be real numbers such that

    9a2 + 8ab+ 7b2 6.Prove that 7a+ 5b+ 12ab 9.

    Proposed by Dr. Titu Andreescu, University of Texas at Dallas

    First solution by Ashay Burungale, India

    Solution. We start from the initial inequality

    9a2 + 8ab+ 7b2 6.By making simple transformations the inequality is equivalent to

    2(a b)2 + 7(a 12

    )2 + 5(b 12

    )2 + 7a+ 5b+ 12ab 9.

    Clearly from this we can derive that

    7a+ 5b+ 12ab

    9.

    Second solution by Daniel Campos Salas, Costa Rica

    Solution. Leta= x +1

    2 and b = y +

    1

    2. The given condition turns

    to

    9x + 9x2 + 4x + 4y+ 8xy+ 7y+ 7y2 = 13x + 9x2 + 11y+ 7y2 + 8xy 0.The result follows from the following chain of inequalities:

    7a+ 5b+ 12ab = 7x+12 + 5y+12 + 12x+12y+12= 9 + 13x+ 11y+ 12xy

    9 + 13x+ 11y+ 8xy+ 4x2 +y2 9 + (13x+ 9x2 + 11y+ 7y2 + 8xy) 9,

    and we are done.

    Mathematical Reflections 1 (2007) 2

  • 8/13/2019 MR 1 2007 Solutions

    3/43

    J33. Consider the sequence: 31, 331, 3331,... whose nth term hasn 3s followed by a 1. Prove that this sequence contains infinitely manycomposite numbers.

    Proposed by Wing Sit, University of Texas at Dallas

    First solution by Jos Alejandro Samper Casas, Colegio Helvetia deBogota, Colombia

    Solution. Let an denote the n-th term of the sequence. We willprove that there exist infinitely many j so that 31|aj.We first note that

    an= 1 +n

    k=1

    3 10k =10n+1 7

    3 .

    Nowa1= 31 = 1027

    3 . Fermats Little Theorem tells us that :

    31|1030 1 = 31|1030k 1 = 31|1030k+2 102

    = 31|1030k+2 102 + (102 7) = 1030k+2 7

    Because gcd(31, 3) = 1 we have that 31|1030k+2

    7

    3 = a30k+1 and we aredone.

    Second solution by Aleksandar Ilic, Serbia

    Solution. Lets compute n-th term:

    an= 33 . . . 33 n

    1 = 1 + 3 10 11 . . . 11 n

    = 1 + 30 10n 19

    =10n+1 7

    3 .

    Its easy to see that all numbers are relatively prime with 2, 3 and5, and that array is strictly increasing. Let p be any prime that dividessome member an of the sequence (we can take p = 31).

    p | an 10n+1 7(modp).From Fermats Little Theorem we get 10p1 1 modp. Now we can

    consider members with indexes of form n +k (p 1), and the problemis solved - because they are composite.

    p | an+k(p1) 10n+1+k(p1) 10n+1

    10p1k 7(modp).

    Also solved by Ashay Burungale, India

    Mathematical Reflections 1 (2007) 3

  • 8/13/2019 MR 1 2007 Solutions

    4/43

    J34. Let ABCbe a triangle and let Ibe its incenter. Prove that atleast one ofIA, IB,IC is greater than or equal to the diameter of theincircle ofABC.

    Proposed by Magkos Athanasios, Kozani, Greece

    First solution by Andrea Munaro, Italy

    Solution. Letr be the inradius of triangleABC. Assume contradic-

    tion, thatI A

  • 8/13/2019 MR 1 2007 Solutions

    5/43

    Second solution by Courtis G. Chryssostomos, Larissa, Greece

    Assume contradiction, that AI

  • 8/13/2019 MR 1 2007 Solutions

    6/43

    Fourth solution by Aleksandar Ilic, Serbia

    Solution. Let projection incenter I on side AB be point C. Fromsine theorem on triangle AICwe have thatAIsin

    2 =r. Analogously,

    we getBIsin 2 =r andC Isin

    2=r. We will prove inequality

    AI BI CI (2r)3,from which at least one ofAI, BI,CI is greater that 2r. Inequality

    is equivalent to

    1 8sin2

    sin

    2sin

    2.

    Function sin x is concave (non-convex) on interval [0, ], so we canuse Jensens inequality and Power Mean:

    sin 2 + sin

    2 + sin

    2

    3 sin

    2 +

    2 +

    2

    3 = sin

    6 =

    1

    2.

    sin 2

    + sin 2

    + sin 2

    3 3sin

    2sin

    2sin

    2.

    Combining above inequalities, the problem is solved.

    Also solved by Daniel Campos Salas, Costa Rica

    Mathematical Reflections 1 (2007) 6

  • 8/13/2019 MR 1 2007 Solutions

    7/43

    J35. Prove that among any four positive integers greater than orequal to 1 there are two, saya and b, such that

    (a2 1)(b2 1) + 1ab

    3

    2

    Proposed by Dr. Titu Andreescu, University of Texas at Dallas

    First solution by Daniel Campos Salas, Costa Rica

    Solution. Let (a,b,c,d) be these numbers. Since these are greaterthan or equal to 1 there exist,,, , in (0, 90], such that

    1

    a,1

    b,1

    c,1

    d

    = (sin , sin , sin , sin ).

    Then,

    Na,b =

    (a2 1)(b2 1) + 1

    ab =

    1 1

    a2

    1 1

    b2

    +

    1

    ab=

    cos cos + sin sin = cos( ).By the Pigeonhole Principle we have that at least two elements of

    the set [,, , ] lie on one of the intervals (0, 30], (30, 60], (60, 90],which implies that there are two of them such that their difference isleast than or equal to 30. Since the function cos x is strictly decreasingon [0, 90], and there exist two elements, say and such that 30 0, we conclude thatNa,b = cos( ) cos30 =

    3

    2 .

    Second solution by Aleksandar Ilic, Serbia

    Solution. Let these four numbers be a,b,c and d. From the condition

    in the problem we can find numbers x, y, z,t [0,

    2 ], such that1

    a= cos x,

    1

    b= cos y,

    1

    c= cos z, and

    1

    d= cos t.

    (a2 1)(b2 1) + 1ab

    =

    1 cos2 x

    1 cos2 y+cos x cos y= cos(xy).By the Pigeonhole Principle, among four numbers x,y,z, t, there are

    two with absolute value less or equal than 6

    . Let|x y| 6

    then

    cos(x y) cos6

    =

    3

    2, and we are done.

    Mathematical Reflections 1 (2007) 7

  • 8/13/2019 MR 1 2007 Solutions

    8/43

    Third solution by David E. Narvaez, Universidad Tecnologica de Panama

    Solution. Let the four numbers be n1, n2, n3 and n4. Since sec reaches every real number greater than or equal to 1 for 0 , thereexist four positive numbers 1,2,3 and 4 such that

    n1 = sec 1, n2= sec 2, n3= sec 3, n4 = sec 4

    and 0

    1, 2, 3, 4 0andf(x)>0, forx 4. Thereforef(x) is a strictly increasing functionand should have at most one solution. We claim thatx= 4 is that uniquesolution and clearly it satisfies the property.

    It is remained to find ifn = 2 and n= 4 satisfy the condition of theproblem.

    Ifn = 2, by Leibnitz Theorem we immediately get that

    P A2 +P B2 +P C2 = 2a2.

    Mathematical Reflections 1 (2007) 16

  • 8/13/2019 MR 1 2007 Solutions

    17/43

    Ifn= 4, (we may assume that Pbelongs to the small arc ofBC) thenusing Ptolemy Theorem and the Law of Cosines we get

    P A= P B+P C, and a2 =P B2 +P C2 +P B P C.Thus

    P A4 +P B4 +P C4 = (P B+P C)4 +P B4 +P C4 =

    = 2(P B4 + 3P B3P C+ 2P B2P C2 + 3PBPC3 +P C4) =

    = 2(P B2

    +P B P C+P C2

    )2

    = 2a4

    .

    Second solution by Aleksandar Ilic, Serbia

    Solution. Assume that circumradius is R = 1. Then the side ofequilateral triangle has length a =

    3. Consider two special positions

    for point P: whenP A and when P A, where A is midpoint forsmaller arc BC. In these cases we have:

    P A: P An +P Bn +P Cn = 0n +ABn +ACn = 2 3n2

    P A: P An

    +P Bn

    +P Cn

    =AAn

    +ABn

    +ACn

    = 2n

    + 2From condition in the problem we have to find all n, such that 3

    n2 =

    2n1 + 1. This can be true only for evenn. By checking, we get thatequality holds for n= 2 and n= 4. In case n6, we have 2n1 + 1 >(

    3)n by trivial induction.

    (

    3)n+1 = (

    3)n

    3< 2n1

    3 +

    3< 2n + 1.

    Now well prove that statement of problem is true in case n = 2 orn= 4. Assume without loss of generality that point P is on smaller arcBC. First apply Ptolemys theorem on quadrilateral ABP C:

    AB P C+AC P B=BC P A P B+P C=P AFrom the Law of Cosines on triangleBC Mwe get

    P B2 +P C2 2P B P C cos 120o =P B2 +P C2 +P B P C=a2.Lets calculate expression for n = 2:

    P A2 +P B2 +P C2 = (P B+P C)2 +P B2 +P C2 =

    = 2(P B2 +P C2 +P B P C) = 2a2 = const.

    Mathematical Reflections 1 (2007) 17

  • 8/13/2019 MR 1 2007 Solutions

    18/43

    In the casen= 4 we get:

    P A4 +P B4 +P C4 = (P B+P C)4 +P B4 +P C4 =

    = 2(P B4 +P C4 + 3P B2 P C2 + 2P B3 P C+ 2P C3 P B) == 2(P B2 +P C2 +P B P C)2 = 2a4 = const.

    Also solved by Courtis G. Chryssostomos, Larissa, Greece;

    David E. Narvaez, Universidad Tecnologica de Panama

    Mathematical Reflections 1 (2007) 18

  • 8/13/2019 MR 1 2007 Solutions

    19/43

    S35. Let ABC be a triangle with the largest angle at A. On theline AB consider the point D such that A lies between B and D and

    AD=AB3

    AC2. Prove that C D

    3 BC

    3

    AC2

    Proposed by Dr. Titu Andreescu, University of Texas at Dallas

    First solution by Ashay Burungale, India

    Solution. Denote AB = c, AC = b, BC = a. If angle A is thegreatest, thena max(b, c). From the Law of Cosines we get

    CD2 =AC2 +AD2 2AC AD cosCAD=

    =b2 +c6

    b4+ 2b c

    3

    b2

    b2 +c2 a22bc

    =

    b6 +c6 +b2c2(b2 +c2 a2)b4

    .

    Thus it is enough to prove that

    b6 +c6 +b2c2(b2 +c2 a2)b4

    3a6

    b4 ,

    b6 +c6 +b2c2(b2 +c2 a2) 3a6,or

    b6 +c6 +b2c2(b2 +c2) 3a6 +a2b2c2.But note that ifa max(b, c), then

    b6 +c6 +b2c2(b2 +c2) b6 +c6 + 2a2b2c2 3a6 +a2b2c2,and we are done.

    Second solution by Daniel Campos Salas, Costa Rica

    Solution. From Stewarts theorem we have

    AD BC2 +AB CD2 =BD(AB AD+AC2), or

    AB

    AB2 BC2

    AC2 +CD2

    = (AB+AD)(AB AD+AC2) =

    AB

    1 +

    AB2

    AC2

    AB4 +AC4

    AC2

    .

    Mathematical Reflections 1 (2007) 19

  • 8/13/2019 MR 1 2007 Solutions

    20/43

    This implies that

    CD2 = (AB2 +AC2)(AB4 +AC4) (AB BC AC)2

    AC4

    = AB6 +AC6 +AB4 AC2 +AB2 AC2(AC2 BC2)

    AC4

    AB6 +AC6 +AB4 AC2

    AC4

    3BC6

    AC4 ,

    that completes the proof.

    Also solved by Aleksandar Ilic, Serbia

    Mathematical Reflections 1 (2007) 20

  • 8/13/2019 MR 1 2007 Solutions

    21/43

    S36. LetPbe a point in the plane of a triangleABC, not lying on thelinesAB,BC,or C A. Denote byAb, Ac the intersections of the parallelsthrough A to the lines P B, P C with the line BC. Define analogouslyBa, Bc, Ca, Cb. Prove thathAb, Ac, Ba, Bc, Ca, Cb lie on the same conic.

    Proposed by Mihai Miculita, Oradea, Romania

    First solution by Ricardo Barosso Campos, Universidad de Sevilla,Spain

    Solution. First of all we remind a famous Carnot Theorem:

    Let ABC be a triangle and points X1, X2, Y1, Y2, Z1, Z2 such thatX1, X2 BC, Y1, Y2 AC, Z1, Z2 AB. If the relation

    AZ1Z1B

    AZ2Z2B

    BX1X1C

    BX2X2C

    C Y1Y1A

    C Y2Y2A

    = 1

    holds, then X1, X2, Y1, Y2, Z1, Z2 lie on a conic.

    We want to prove that Ab, Ac, Ba, Bc, Ca, Cb satisfy this condition.

    LetAP

    BC=Ap , BP

    AC=Bp , CP

    AB= Cp.

    DenoteAB=c, BC=a, CA= b and

    BAp = m, ApC=n, CBp = q, BpA= r, ACp = s, CpB = t.

    Using the similarity of trianglesBCpCand BAACwe get

    BAcBC

    = BA

    BCP, BAC= ca

    t .

    Analogously,

    BCa=

    ca

    m , CAB =

    ba

    q , CBa =

    ab

    n, ACb=

    cb

    r, ABc=

    bc

    s.

    On the other hand we have

    BAb= CAb a= baq a= a(b q)

    q =

    ar

    q .

    Similarly,

    BCb=cq

    r, CAc=

    as

    t , CBc=

    bt

    s, ABa=

    bm

    n , ACa=

    cn

    m.

    Mathematical Reflections 1 (2007) 21

  • 8/13/2019 MR 1 2007 Solutions

    22/43

    Finally we calculate the product

    ACaBCa

    ACbBCb

    BAcCAc

    BAbCAb

    C BaABa

    C BcABc

    =

    =

    cn/m

    ca/m cb/r

    cq/r

    ca/t

    as/tar/q

    ba/q

    ab/n

    bm/n bt/s

    bc/s

    =

    =

    n

    a b

    q

    c

    sr

    b

    a

    mt

    c

    =

    n r tq

    s

    m

    = 1.

    Last equality follows from Ceva Theorem, and we are done.

    Mathematical Reflections 1 (2007) 22

  • 8/13/2019 MR 1 2007 Solutions

    23/43

    Second solution by Mihai Miculita, Oradea, Romania

    Solution. Denote{Pa} = AP BC,{Pb} = BP AC,{Pc} =P C AB, and with

    {Qa} =BaCa BC, {Qb} =AbCb AC, {Qc} =AcBc AB.If (l ,m,n) are barycentric coordinates of point P with respect to

    triangleABC, then pointsPa, Pb, Pc have coordinates:

    P(0, m , n), Pb(l, 0.n), Pc(l,m, 0).

    Observe the following facts

    AAb||P B, AbBAbC

    =APbAC

    ; PbA

    PbC =

    n

    l, APb

    AC =

    n

    l+n.

    ThereforeAbB

    AbC =

    n

    l+n, Ab(0, l+n, n).

    Analogously, we get

    Ac(0,

    m, l+m), Ba(m+n, 0,

    n), Bc(

    l, 0, l+m), Ca(m+n,

    m, 0), Cb(

    l, l+n, 0).

    The equation of the line BaCa is

    x y z

    m+n 0 nm+n m 0

    = 0, mn x+n(m+n) y+m(m+n) z= 0.ThusQa is the solution of the system

    x= 0 ; mn x+n(m+n) y+m(m+n) z= 0.It follows that y

    m z

    n, Qa= (0,m,n).Similarly we get the coordinates of the points Qb(l, 0, n) and Qc(l, m, 0).Because the determinant

    0 m nl 0 nl m 0

    = 0,we deduce that points Qa, Qb, Qc are collinear. Using the reciprocal ofthe Pascal Theorem, we get that points Ab, Ac, Ba, Bc, Ca, Cb lie on thesame conic.

    Mathematical Reflections 1 (2007) 23

  • 8/13/2019 MR 1 2007 Solutions

    24/43

  • 8/13/2019 MR 1 2007 Solutions

    25/43

  • 8/13/2019 MR 1 2007 Solutions

    26/43

    U32. Let a0, a1, . . . , an and b0, b1, , bn be sequences of complexnumbers. Prove that

    Re

    nk=0

    akbk

    1

    3n+ 2

    nk=0

    |ak|2 +9n2 + 6n+ 2

    2

    nk=0

    |bk|2

    Proposed by Jose Luis Daz-Barrero, Barcelona, Spain

    First solution by Aleksandar Ilic, Serbia

    Solution. Letak =xk+ yk and bk =Xk Yk, for 0k n. Wecan assume thatxk, yk, Xk, Yk 0, because we can increase left hand-sideusing absolute values. Now we have to prove inequality:

    nk=0

    (xkXk+ykYk) 13n+ 2

    nk=0

    x2k+y

    2k

    +

    9n2 + 6n+ 2

    2

    nk=0

    X2k +Y

    2k

    .

    Because of symmetry we only need to prove:

    2(3n+ 2)xkXk

    2x2k+ (9n2 + 6n+ 2)X2k

    When consider this as quadratic inequality for variable xk, the dis-criminant is negative, and thus parabola is above y = 0.

    = (2(3n+ 2)Xk)2 4 2 (9n2 + 6n+ 2)X2k =

    = 4X2k

    9n2 + 12n+ 4 18n2 12n 4 = 36n2X2k

  • 8/13/2019 MR 1 2007 Solutions

    27/43

    This can be separated into

    0 13n+ 2

    x2 +

    9n2 + 6n+ 2

    2 w2

    xw, (1)

    and

    0 13n+ 2

    y2 +

    9n2 + 6n+ 2

    2 z2

    +yz. (2)

    We have that (1) equals

    (x w)2 + (x (3n+ 1)w)22(3n+ 2)

    ,

    and (2) equals

    (y+z)2 + (y+ (3n+ 1)z)2

    2(3n+ 2) .

    These inequalities prove our claim and complete the proof.

    Mathematical Reflections 1 (2007) 27

  • 8/13/2019 MR 1 2007 Solutions

    28/43

  • 8/13/2019 MR 1 2007 Solutions

    29/43

    U34. Let f : [0, 1] R be a continuous function with f(1) = 0.Prove that there is a c (0, 1) such that

    f(c) =

    c0

    f(x)dx

    Proposed by Cezar Lupu, University of Bucharest, Romania

    Solution by Aleksandar Ilic, Serbia

    Solution. Let F(x) =x0

    f(t)dt and F(x) = f(x) and F(0) =f(1) = 0. Consider continuous function g(x) = F(x) f(x). Withoutloss of generality, assume that g(x)> 0 forx (0, 1). Forx = 0 we haveg(0) = f(0) 0, and forx = 1 we have g(1) =F(1) 0.

    Leth(x) =exF(x). We can compute first derivative for h(x):

    h(x) = exF(x) +exf(x) = ex(F(x) f(x)) = exg(x) 0

    So, functionh is decreasing function and h(0) = 0 and h(1) = F(1)e

    0. This gives h(x) = 0, and contradiction! Thus there is c(0, 1) suchthatg(c) = 0 or equivalently f(c) =F(c).

    Mathematical Reflections 1 (2007) 29

  • 8/13/2019 MR 1 2007 Solutions

    30/43

    U35. Find all linear maps f :Mn(C) Mn(C) such thatf(XY) =f(X)f(Y) for all nilpotent matrices X andY.

    Proposed by Gabriel Dospinescu, Ecole Normale Superieure, Paris

    Solution by Jean-Charles Mathieux, Dakar University, Senegal

    Solution. We consider Ka zero characteristic field and do not limitthe answer to C.

    Denote byfa linear mapf :Mn(K) Mn(K) such thatf(XY) =f(X)f(Y) for all nilpotent matrices X and Y . We show that eitherf = 0 or there exists P GLn(K) such that f(M) = P MP1 for allM Mn(K).

    The casef= 0 is obvious so lets assume that f= 0.Denote by Ei,j the matrix ofMn(K) with all coefficients equal to 0

    except on row i and column j, where it equals 1 and Fi,j =f(Ei,j).

    We proceed in two steps.

    Lemma 1. {Fi,j}i,j{1,...,n} is a basis ofMn(K) such that Fi,jFk,l =j,kFi,l.

    Lemma 2. If{Mi,j}i,j{1,...,n}is a basis ofMn(K) such thatMi,jMk,l =j,kMi,l, then there exists P GLn(K) such thatMi,j =P Ei,jP1 for alli, j {1, . . . , n}.

    Proof of Lemma 1. If{Fi,j}i,j{1,...,n} was not a basis ofMn(K),we could find{i,j}i,j{1,...,n} Kn2 with at least one l,p = 0 and

    i,j{1,...,n} i,jFi,j = 0

    For all i, j, k,l {1, . . . , n} we have Ei,jEk,l = j,kEi,l so Fi,jFk,l =j,kFi,l. This last relation is obvious if i = j and k = l, becauseEi,j and Ek,l are nilpotent. But also f(Ei,iEk,l) = f(Ei,j

    0

    Ej0,iEk,l) =

    f(Ei,j0i,kEj0,l) =i,kFi,l (where we choose j0=i and j0=l).There is at least one Fk,q= 0 because f= 0,Fk,l

    i,j{1,...,n} i,jFi,j

    Fp,q =l,pFk,q = 0 which is a contradiction, so{Fi,j}i,j{1,...,n} is a basisofMn(K).

    Proof of Lemma 2. Mi,kMk,j =Mi,j , so ImMi,k = ImMi,j , lets denotethis space byVi.

    There exist{i,j}i,j{1,...,n} Kn2 such thatIn=

    i,j{1,...,n} i,jMi,j .If X Kn, X =ni=1nj=1 i,jMi,jX, andnj=1 i,jMi,jX Vi, soKn = V1+ V2+ + Vn.

    Mathematical Reflections 1 (2007) 30

  • 8/13/2019 MR 1 2007 Solutions

    31/43

    IfXi Vi, andX1 + X2 + + Xn= 0 thenMi,i(X1 + X2 + Xn) =Xi = 0, so K

    n = V1 V2 V n.Ifk=j , ImMi,k KerMi,j , so

    nk=1,k=jImMi,k = KerMi,j (because

    they have same dimension and the previous inclusion).Let Y1 V1, such that Y1= 0. For i= 1, Yi = Mi,1Y1. We have

    Mi,jYj =Y1. So{Y1, . . . , Y n} is a basis ofKn.We define P GLn(K) the matrix whose columns are the Yi. We

    have P Ei,j =Mi,jP, which concludes the proof of lemma 2.

    Since f is linear we have proven that f(M) = P MP1

    for all MMn(K).

    Mathematical Reflections 1 (2007) 31

  • 8/13/2019 MR 1 2007 Solutions

    32/43

    U36. Let n be an even number greater than 2. Prove that if thesymmetric group Sn contains an element of order m, then GLn2(Z)contains an element of order m.

    Proposed by Jean-Charles Mathieux, Dakar University, Senegal

    No solutions proposed

    Mathematical Reflections 1 (2007) 32

  • 8/13/2019 MR 1 2007 Solutions

    33/43

    Olympiad

    O31. Letn is a positive integer. Prove that

    nk=0

    n

    k

    n+k

    k

    =

    nk=0

    2k

    n

    k

    2

    Proposed by Jean-Charles Mathieux, Dakar University, Senegal

    First solution by Zhao Bin, HUST, China

    Solution. We start from the right side of the equality. First of all,because

    2k =ki=0

    k

    i

    We have

    n

    k=02k

    n

    k2

    =n

    k=0k

    i=0 k

    in

    k2

    =n

    i=0n

    k=i k

    in

    k2

    .

    Then using well-known identitiesk

    i

    n

    k

    =

    n

    i

    n in k

    and

    nk=i

    n in k

    n

    k

    =

    2n i

    n

    .

    The first inequality goes directly from the binomial theorem, while thesecond can be obtained from the combinational way or comparing the

    the coefficient ofxn in the identity:

    (1 +x)ni(1 +x)n = (1 +x)2ni.

    Thus we haven

    k=0

    2k

    n

    k

    2=

    ni=0

    nk=i

    k

    i

    n

    k

    2=

    ni=0

    n

    i

    nk=i

    n in k

    n

    k

    =

    =ni=0

    n

    i

    2n i

    n

    .

    Mathematical Reflections 1 (2007) 33

  • 8/13/2019 MR 1 2007 Solutions

    34/43

    Then by substituting k = n i, we easily getni=0

    n

    i

    2n i

    n

    =

    nk=0

    n

    k

    n+k

    k

    .

    Thus we have proved the desired identity.

    Second solution by Jose Hernandez Santiago, UTM, Oaxaca, Mexico.

    Solution. Let m N with n m. The desired identity is a specialcase of the more general result (see Wilf, Herbert S. Generatingfunc-tionology. p. 127),

    mk=0

    m

    k

    n+k

    m

    =

    mk=0

    2k

    m

    k

    n

    k

    . (3)

    Settingm = n in (1) we get,

    n

    k=0 2k

    n

    k2

    =n

    k=0 2k

    n

    kn

    k=

    nk=0

    n

    k

    n+k

    n

    =n

    k=0

    n

    k

    n+k

    n+k n

    =n

    k=0

    n

    k

    n+k

    k

    .

    Next two solutions were kindly presented by Dr. Scott H. Brown,Auburn University Montgomery.

    Third solution (Benjamin and Bataille)

    Solution. Let [n] denote the set {1, 2, 3, . . . , n} andD denote the setof ordered pairs (A, B) whereA is a subset ofn andB is an n-subset of[2n] that is disjoint from A. We can select elements for D in two ways:

    Mathematical Reflections 1 (2007) 34

  • 8/13/2019 MR 1 2007 Solutions

    35/43

  • 8/13/2019 MR 1 2007 Solutions

    36/43

    O32. 18. Let a, b, c >0. Prove that a2

    4a2 +ab+ 4b2+

    b2

    4b2 +bc+ 4c2+

    c2

    4c2 +ca+ 4a2 1

    Proposed by Bin Zhao, HUST, China

    First solution by Ho Phu Thai, Da Nang, Vietnam

    Solution. By Cauchy-Schwarz inequality a2

    4a2 +ab+ 4b2+

    b2

    4b2 +bc+ 4c2+

    c2

    4c2 +ca+ 4a2

    (8a2 + 8b2 + 8c2 +ab+bc+ca)

    cyc

    a2

    (4a2 +ab+ 4b2)(4a2 +ac+ 4c2).

    We will prove

    cyc

    a2

    (4a2 +ab+ 4b2)(4a2 +ac+ 4c2) 1

    8a2 + 8b2 + 8c2 +ab+bc+ca .

    This is equivalent to

    64sym

    a4b2 + 4sym

    a4bc+ 4sym

    a3b3 + 3sym

    a3b2c 66a2b2c2,

    which is clearly true from AM-GM inequality.

    Second solution by Vasile Cartoaje, University of Ploiesti, Romania

    Solution. By the Cauchy-Schwarz inequality we have a2

    4a2 +ab+ 4b2+

    b2

    4b2 +bc+ 4c2+

    c2

    4c2 +ca+ 4a2

    a2(4a2 +ab+ 4b2)(4c2 +ca+ 4a2)

    (4c2 +ca+a2) =

    =

    a2(4b2 +bc+ 4c2) (4c2 +ca+ 4a2)

    (4a2 +ab+ 4b2)(4b2 +bc+ 4c2)(4c2 +ca+ 4a2)=

    A

    B,

    Mathematical Reflections 1 (2007) 36

  • 8/13/2019 MR 1 2007 Solutions

    37/43

    whereA= (8

    b2c2 +abc

    a)(8

    a2 +

    bc),

    B= (4a2 +ab+ 4b2)(4b2 +bc+ 4c2)(4c2 +ca+ 4c2).

    It suffices to prove thatA B. We have

    A= 64(

    a2)(

    b2c2) + 8(

    bc)(

    b2c2)+

    +8abc( a2)( a) +abc( bc)( a) == 64

    b2c2(b2+c2)+195a2b2c2+17abc

    bc(b+c)+8abc

    a3+8

    b3c3.

    and

    B= 64

    (b2+c2)+16

    bc(a2+b2)(c2+a2)+4abc

    a(b2+c2)+a2b2c2 =

    = 64

    b2c2(b2+c2)+129a2b2c2+20abc

    bc(b+c)+16abc

    a3+16

    b3c3.

    Therefore

    A B= 3abc bc(b+c) + 8abc a3 + 8 b3c3 66a2b2c2 == 3abc[

    bc(b+c)6abc]+8abc(

    a33abc)+8(

    b3c33a2b2c2) 0,

    because bc(b+c) 6abc=

    a(b c)2 0,

    and by the AM-GM inequality,a3 3abc,

    b3c3 3a2b2c2.

    The equality occurs if and only ifa = b = c.

    Mathematical Reflections 1 (2007) 37

  • 8/13/2019 MR 1 2007 Solutions

    38/43

    O33. 23. Let ABCbe a triangle with cicrumcenter O and incenterI. Consider a point Mlying on the small arc BC. Prove that

    MA+ 2OI MB+M C MA 2OIProposed by Hung Quang Tran, Ha Noi University, Vietnam

    Solution by Hung Quang Tran, Ha Noi University, Vietnam

    Solution. Using the Law of Cosines in the triangle AOM we get2AO OM cos AMO =M A2 +AO2 MO2 =M A2, 2MO MA=MA2, Analogously

    2MO MA= M B2, 2MO MC=M C2

    From here we have

    MB+MC MA= 2MO

    MA

    MA+

    M B

    MB+

    M C

    MC

    From the Cauchy- Scwartz Inequality we get

    MO MAMA

    + M BMB

    +M CMC

    2MO MAMA

    + M BMB

    +M CMC

    MO MA

    MA+

    M B

    MB+

    M C

    MC

    We have M O= R, we will calculate MA

    MA+

    M B

    MB+

    M C

    MC .

    MA

    MA+

    MB

    MB+

    MC

    MC2 = 3+2

    MB MCMB MC +

    M B MAMB MA+

    MA MAMC MA

    =

    = 3 + 2(cos(MB,MC) cos(MB,MA) cos(M C , M A)) =3 2(cos A+ cos B+ cos C) = 3 2 R +r

    R =

    OI2

    R2 .

    Thus we have

    MO MAMA

    +M B

    MB+

    M C

    MC =OI.

    It follows that

    MA+ 2OI MB+M C MA 2OI ,and the problem is solved.

    Mathematical Reflections 1 (2007) 38

  • 8/13/2019 MR 1 2007 Solutions

    39/43

  • 8/13/2019 MR 1 2007 Solutions

    40/43

    h Z[X]. If we take x = m such that f(m2) is a perfect square, weget N

    f(m2) = h(m). This relations holds for infinitely manym thus

    f(x2) = g(x)2. As g has no non-zero roots, its roots must group intopairs of opposite roots (because the roots off(x2) do). Thusg(x) =r(x2)so f(x) =r2(x), and clearly r(x) Z[X] (we can use lemma 1 to prover(x) Q[X] then we deducer(x) Z[x]). Now let us solve the problem.

    Letf(x) =P(x) =

    n

    i=1(x wn), then P(x2 +ax) = n

    i=1(y2 ui),

    where ui =

    a2

    4 +wi and y = x+

    a

    2 . Thus Q(x) =ni=1(x2 ui) isirreducible. The Q(x) can be written as f(x) g(x) wheref(x) =u(x2)h(x), g(x) = v(x2)h(x).

    From all such representations f(x) g(x), pick up one for which deg(f)is the least possible. It follows that gcd(f(x), g(x)) is either 1 orf(x), which means that either h(x) = 1 or u(x)|v(x). Ifu(x)|v(x) andu= 1, then gcd(f(x), g(x))= 1. Thus gcd(f(x), g(x)) = f(x), hencef(x)2|Q(x). This readily implies thatPhas a double root (ifa is suffi-ciently big), so (P, P) = 1 and therefore Pis irreducible. It follows thatwe have h(x) = 1 or u(x) = 1.

    a) If h(x) = 1, then f(x) = u(x2), g(x) = v(x2), u(x)v(x) =f(x+ a2) so f is reducible.

    b) Ifu(x) = 1,then f(x) =h(x), g(x) =h(x)v(x2), h(x)h(x)v(x2) = Q(x). If h(x)h(x) = w(x2) we get w(x)v(x) =f(x + a2), thusf is reducible unlessv = 1. In this case we have Q(x) =h(x)h(x). ThereforeQ(0) =h(0)2 and thusP(a2

    4) is a perfect square.

    By setting k(x) = 4mf(x4

    ) for a suitable m, the conditions of lemma 2are satisfied. Hence k is a perfect square, so must be P.

    In any case we get that f=P is reducible.

    Remark. The key assertion that iff Z[X] is irreducible and f(0)is not a perfect square thenf(x2) is also irreducible was a problem fromRomanian TST 2003.

    Mathematical Reflections 1 (2007) 40

  • 8/13/2019 MR 1 2007 Solutions

    41/43

    O35. Let 0 < a < 1. Find, with proof, the greatest real numberb0such that ifb < b0 and (An [0;1])nN are finite unions of disjoint seg-ments with total lengtha, then there are two different i, j Nsuch thatAi

    Aj is a union of segments with total length at least b. Generalizethis result to numbers greater than 2: if k N find the least b0 suchthat whenever b < b0 and (An [0;1])nN are finite unions of disjointsegments with total lengtha, then there are k differenti1, i2, . . . , ik Nsuch that Ai1

    Ai2 . . .

    Aik is a union of segments with total length

    at leastb.

    Proposed by Iurie Boreico, Moldova

    Solution by Iurie Boreico, Moldova

    Solution. We will use the following famous result:n

    k

    >

    nk k2nk1k!

    Indeed,n

    k

    = n(n1)...(nk+1)

    k! . Using the following inequalityxy (x+ 1)(y 1), forx > y,we conclude that

    n(n 1) . . . (n k+ 1) > nk1(n 1 2 . . . (k 1)),from which we deduce the result.

    Let us solve the general problem. Pick up some n. Suppose [0, 1] is di-vided into intervals of lengths x1, x2, x3, . . . , xmby the sets A1, A2, . . . , An.Letxi belong toki ofA1, A2, . . . , An, andk1x2+ k2x2+ . . . + knxn= na.

    Suppose that each k or the Ai intersect in a set of measure at mostb. Consider the sum

    1i1

  • 8/13/2019 MR 1 2007 Solutions

    42/43

    which translates into

    mi=1

    kki xi (mi=1

    kixi)k =nkak.

    Therefore the sum in contest is at least nk

    k!(ak k2

    n). But weve prove it is

    at mostbn

    k

    < b n

    k

    k!. Thus ifb < ak, we get a contradiction for sufficiently

    bign.

    Let us prove that any number greater than ak fails. Indeed, considerb > ak and pick up a rational number p

    qsuch thata < p

    q < k

    b. Consider

    Ai be the set of all real numbers from [0; 1] which written in base qhavethe i-th digit after zero from the set{0, 1, . . . , p 1}. Its obvious thatAi are a union of segments, that|Ai| = pq > a and|Ai1

    Ai2. . . Aik | =

    (pq

    )k < b so this is the required counter-example.

    The answer is thus ak.

    Mathematical Reflections 1 (2007) 42

  • 8/13/2019 MR 1 2007 Solutions

    43/43

    O36. Leta1, a2,..,an and b1, b2,...,bn be real numbers and let xij bethe number of indicesk such thatbk max(ai, aj). Suppose thatxij >0for any i and j. Prove that we can find an even permutationf and an

    odd permutationg such thatni=1

    xif(i)xig(i)

    n.

    Proposed by Gabriel Dospinescu, Ecole Normale Superieure, Paris

    Solution by Iurie Boreico, Moldova

    Solution. Ifn = 2 then we must prove that x1,1+ x2,2 2x1,2 whichis true because clearly x1,1 x1,2 andx2,2 x1,2.

    Now assume n 3. By AM-GM it suffices to find an even permuta-tionfand an odd permutation g such that

    ni=1

    xi,f(i)ni=1

    xj,g(j).

    Setxi() =xi,(i) for a permutation .We must find an even permutationfand an odd permutationg such

    thatni=1 xi(f) ni=1 xi(g). If X() =ni=1 xi() we must haveX(f) X(g). This will follow directly from the relationfX(f) =

    gX(g) where f runs over all even permutations and g over all oddpermutations (recall that the number od even permutation equals thenumber of odd permutations).

    To prove this relation, it suffices to prove that

    fxi(f) =

    gxi(g).This fact follows from the following result:

    For any 1 i, j n, the number of even permutations f such thatf(i) =j equals the number of odd permutations g such thatg(i) =j.

    Indeed if we pick up k, l such that j, k, l are pairwise different, thenthe permutations with(i) =j group into pairs , () where is the

    transposition exchanging k with l. As , () have different signs, wededuce there is one even permutation and one odd permutation in eachpair, hence the result.